Difference between revisions of "Mock AIME 1 2010 Problems/Problem 3"

(Created page with "Lo wassup homies")
 
m (page was empty, so i added a see also section, statement of the problem, and statement of the answer)
Line 1: Line 1:
Lo wassup homies
+
== Problem ==
 +
Let <math>AB</math> be a line segment of length <math>20 \sqrt{2}</math>, and let <math>S</math> be the set of all points <math>P</math> such that <math>m \angle APB \geq 45^{\circ}</math>. Find the last three digits of the largest integer less than the area of <math>S</math>.
 +
 
 +
== Solution ==
 +
<math>\boxed{284}</math>.
 +
 
 +
== See Also ==
 +
*[[Mock AIME 1 2010 Problems]]
 +
*[[Mock AIME 1 2010 Problems/Problem 2|Followed by Problem 2]]
 +
*[[Mock AIME 1 2010 Problems/Problem 4|Followed by Problem 4]]

Revision as of 07:51, 2 August 2024

Problem

Let $AB$ be a line segment of length $20 \sqrt{2}$, and let $S$ be the set of all points $P$ such that $m \angle APB \geq 45^{\circ}$. Find the last three digits of the largest integer less than the area of $S$.

Solution

$\boxed{284}$.

See Also